Propagator for a free particle / schrodinger equation

In summary, a propagator for a free particle is a mathematical function derived from the Schrodinger equation that describes the probability of a particle's movement without external forces. It is related to the Schrodinger equation and is crucial in understanding the behavior of quantum systems. It is applicable to all particles and plays a significant role in many quantum mechanical calculations and experiments.
  • #1
boboYO
106
0
http://img18.imageshack.us/img18/4295/eqn.png here is the text preceding the exercise:

http://yfrog.com/5mch5p in the exercise, where does the factor [tex]\frac{m}{(2mE)^{1/2}}[/tex] come from? Comparing that equation with 5.19 (bottom right of link), why can't we just replace |p> with |E,+> and |E,->?

if I start from scratch:

[tex]| \psi \rangle = \sum_{\alpha=\pm} \int_0^{\infty}\! |E,\alpha\rangle\langle E,\alpha|\psi \rangle \,\,dE [/tex]do [tex]\left( i \hbar \frac{\partial}{\partial t} - H \right)[/tex] to both sides:

[tex] \left( i \hbar \frac{\partial}{\partial t} - H \right) |\psi \rangle = 0 = \sum_{\alpha=\pm} \int_0^{\infty}\!\left[i \hbar \langle E,\alpha|\dot{\psi}\rangle -E\langle E\langleE,\alpha|\psi\rangle \right] |E, \alpha \rangle \,\, dE [/tex]

[tex]\implies i \hbar \langle E,\alpha|\dot{\psi}\rangle -E\langle E\langleE,\alpha|\psi\rangle = 0 [/tex]

[tex] \implies \langle E,\alpha| \psi \rangle = \langle E,\alpha|\psi_0\rangle e^{-iEt/\hbar}[/tex][tex]\therefore | \psi \rangle = \sum_{\alpha=\pm} \int_0^{\infty}\! |E,\alpha\rangle \langle E,\alpha|\psi_0\rangle e^{-iEt/\hbar} \,\,dE [/tex]and so the propagator is [tex] U(t) = \sum_{\alpha=\pm} \int_0^{\infty}\! |E,\alpha\rangle \langle E,\alpha| e^{-iEt/\hbar} \,\,dE [/tex]

??
 
Last edited by a moderator:
Physics news on Phys.org
  • #2
Split (5.1.9) into two integrals, one for positive p and one for negative p, and when you change variables from p to E, remember that you have to express dp in terms of dE.
 
  • #3
Thanks. I mistakenly assumed dp=dE because |p>=|E>but I still can't find the mistake in my derivation? Is the first line correct?
 
  • #4
Just a guess, but maybe it's a just difference in normalization.
 
  • #5
It can not be a just difference in normalization, since [tex]\frac{m}{(2mE)^{1/2}}[/tex] is the factor of the integrand. I am facing the same problem, any help is appreciated.
 
  • #6
That factor is necessary as a measure to properly count the density of states. The point is that the number of states in the interval [tex](E,E+dE)[/tex] must be the same as the number of states in the interval [tex](p,p+dp)[/tex] when [tex]E = p^2/(2m)[/tex].

For a free particle, the momentum states are parameterized as a linear function of wavenumbers, so it's obvious that

[tex]\sum_p \rightarrow \int dp[/tex]

without any factors. If we compute

[tex]\int_{\sqrt{2mE}}^{\sqrt{4mE}} = (\sqrt{2}-1) \sqrt{2mE} \neq \int_E^{2E} dE.[/tex]

However

[tex] \int_E^{2E} dE \frac{m}{\sqrt{2mE}} = (\sqrt{2}-1) \sqrt{2mE}.[/tex]

We therefore conclude that

[tex]\sum_E \rightarrow \int dE ~\frac{m}{\sqrt{2mE}}.[/tex]
 
  • #7
I do not understand why the number of states in the two interval must be same, through it leads to the desired result. But the point is, what is wrong with the derivation above? Isn't that the normal form of propagator?
 
  • #8
kerrigan said:
I do not understand why the number of states in the two interval must be same, through it leads to the desired result. But the point is, what is wrong with the derivation above? Isn't that the normal form of propagator?

It is, but you still have to change the momentum integration to an energy integration, which introduces the same factor. Start with

[tex] U(t) = \int_{-\infty}^\infty dp | p \rangle \langle p | e^{-iEt/\hbar}[/tex]

We can introduce complete sets of energy eigenstates:

[tex] U(t) = \int_{0}^\infty dp \int_0^\infty dE' \int_0^\infty dE'' |E',+\rangle\langle E'',+ | \langle E'',+ | p \rangle \langle p | E',+ \rangle e^{-iEt/\hbar}
+ (+\rightarrow -).[/tex]

Now we know that

[tex]\langle p | E',+\rangle = \langle E=p^2/2m, + | E',+\rangle = \delta(E-E'),[/tex]

but to use that we'll need to rewrite the integral over momentum in terms of an integral over energy:

[tex]\int_0^\infty dp = \int_0^\infty \frac{m}{\sqrt{2mE}} dE.[/tex]
 
  • #9
Thank you for your reply. I am puzzled with what you "start with". We are dealing with the free particle, so [tex]H=\frac{m}{(2mE)^{1/2}} [/tex]. If a state is an eigenstate of [tex]H[/tex], it should be an eigenstate of [tex]P^2[/tex], and thus an eigenstate of [tex]P[/tex]. (Why? We know that any eigenstate of [tex]P[/tex] is also eigenstate of [tex]P ^2[/tex], but vice versa?) As the result, an eigenstate of energy can be labeled as |p>.

The normal form of propagator is [tex]
U(t) = \int_{0}^\infty dE | E \rangle \langle E | e^{-iEt/\hbar}
[/tex].
Then replace |E> with |p>, it seems that the propagator should be
[tex]
U(t) = \int_{0}^\infty dE | p \rangle \langle p | e^{-iEt/\hbar}
[/tex]. What's wrong?

I'm reading shankar. If you have a book at hand, you may see p.146 and p.152. Thanks.
 
  • #10
kerrigan said:
Thank you for your reply. I am puzzled with what you "start with". We are dealing with the free particle, so [tex]H=\frac{m}{(2mE)^{1/2}} [/tex].

That's not the free particle Hamiltonian. Among other things, that expression has the wrong units!.

If a state is an eigenstate of [tex]H[/tex], it should be an eigenstate of [tex]P^2[/tex], and thus an eigenstate of [tex]P[/tex]. (Why? We know that any eigenstate of [tex]P[/tex] is also eigenstate of [tex]P ^2[/tex], but vice versa?)

The vice versa comes from the theorem that if two observables commute, they have simultaneous eigenstates.

As the result, an eigenstate of energy can be labeled as |p>.

The normal form of propagator is [tex]
U(t) = \int_{0}^\infty dE | E \rangle \langle E | e^{-iEt/\hbar}
[/tex].
Then replace |E> with |p>, it seems that the propagator should be
[tex]
U(t) = \int_{0}^\infty dE | p \rangle \langle p | e^{-iEt/\hbar}
[/tex]. What's wrong?

I'm reading shankar. If you have a book at hand, you may see p.146 and p.152. Thanks.

This thread was about a particular definition of the propagator that is equation 5.1.9 in the link in the OP. It is more common to define the propagator as a sum over energy eigenstates. However, if you rewrite your last expression above as an integral over momentum, you'll find an extra factor of [tex]p/m[/tex]. So the two definitions of propagator are not perfectly in agreement, but I believe that the qualitative physics will be the same.
 

Related to Propagator for a free particle / schrodinger equation

What is a propagator for a free particle?

A propagator for a free particle is a mathematical function that describes the probability of a particle moving from one point in space to another without any external forces acting upon it. It is an essential concept in quantum mechanics and is derived from the Schrodinger equation.

What is the Schrodinger equation?

The Schrodinger equation is a fundamental equation in quantum mechanics that describes the time evolution of a quantum system. It is used to determine the wave function of a particle and to predict its behavior over time.

How is the propagator for a free particle related to the Schrodinger equation?

The propagator for a free particle is derived from the Schrodinger equation by solving for the wave function of a particle moving in free space. It is a solution to the time-dependent Schrodinger equation and can be used to calculate the probability of a particle being found at a certain position and time.

What is the significance of the propagator for a free particle?

The propagator for a free particle is essential in calculating the behavior of quantum systems. It is used to predict the future state of a particle and to understand its movement and interactions with its surroundings. It also plays a crucial role in many quantum mechanical calculations and experiments.

Is the propagator for a free particle applicable to all particles?

Yes, the propagator for a free particle can be used for any type of particle, including electrons, protons, and even larger objects. It is a fundamental concept in quantum mechanics and is applicable to all quantum systems.

Similar threads

  • Advanced Physics Homework Help
Replies
3
Views
915
  • Advanced Physics Homework Help
Replies
10
Views
643
Replies
16
Views
633
  • Advanced Physics Homework Help
Replies
9
Views
1K
Replies
4
Views
1K
  • Advanced Physics Homework Help
Replies
7
Views
1K
  • Advanced Physics Homework Help
Replies
1
Views
1K
  • Advanced Physics Homework Help
Replies
8
Views
3K
  • Advanced Physics Homework Help
Replies
29
Views
238
Replies
7
Views
2K
Back
Top